La existencia de los núcleos depende de una serie de condiciones de contorno de la mecánica cuántica. Aparecen como soluciones a un problema en el que hay un equilibrio de: a) la fuerza de color atractiva que une a los quarks en un protón o un neutrón, b) la fuerza electromagnética repulsiva entre protones, c) el principio de exclusión de Pauli, d) la inestabilidad de los neutrones no fuertemente unidos a una desintegración débil. Hay factores adicionales que entran una vez que los electrones quedan atrapados alrededor de un núcleo, pero esa es otra historia.
Para responder al "por qué" del elemento con 26 protones y 30 neutrones es estable (o el que tiene 26 protones y 32 neutrones) y se acerca a la energía de enlace máxima, se necesita un modelo mecánico cuántico específico para el potencial colectivo de los factores mencionados. Modelos de concha tienen bastante éxito en la clasificación de la tabla periódica.
Sin embargo, la verdadera respuesta sobre el hierro sería fenomenológica, es decir, lo que observamos y encaja fenomenológicamente con el Weizsaecker que se basa en un modelo de gota de líquido. Según el funcionamiento del potencial efectivo, la inclusión de más y más nucleones en el pozo de potencial después del hierro deja de crear un pozo de potencial efectivo más profundo, debido al aumento del efecto de las fuerzas repulsivas descritas anteriormente.
Tenga en cuenta que es el Ni62 el que está más unido en el curva de energía de enlace.
3 votos
El Fórmula de Bethe-Weizsäcker ofrece alguna explicación para la energía de enlace nuclear. Sin embargo, el motivo por el que se alcanza el mínimo para el hierro depende de los parámetros de la fórmula de Bethe-Weizsäcker, que a su vez dependen de las particularidades de las fuerzas entre los nucleones y preguntarse por qué son, cómo son, no es físico.
0 votos
Relacionado: physics.stackexchange.com/q/182525/2451 y los enlaces que contiene.
0 votos
Leer sobre los niveles de estabilidad y de núcleo en Wikipedia : Modelo de cáscara nuclear
0 votos
Ver esto:- chemistry.stackexchange.com/questions/40407/